Quantcast
  • Register
PhysicsOverflow is a next-generation academic platform for physicists and astronomers, including a community peer review system and a postgraduate-level discussion forum analogous to MathOverflow.

Welcome to PhysicsOverflow! PhysicsOverflow is an open platform for community peer review and graduate-level Physics discussion.

Please help promote PhysicsOverflow ads elsewhere if you like it.

News

PO is now at the Physics Department of Bielefeld University!

New printer friendly PO pages!

Migration to Bielefeld University was successful!

Please vote for this year's PhysicsOverflow ads!

Please do help out in categorising submissions. Submit a paper to PhysicsOverflow!

... see more

Tools for paper authors

Submit paper
Claim Paper Authorship

Tools for SE users

Search User
Reclaim SE Account
Request Account Merger
Nativise imported posts
Claim post (deleted users)
Import SE post

Users whose questions have been imported from Physics Stack Exchange, Theoretical Physics Stack Exchange, or any other Stack Exchange site are kindly requested to reclaim their account and not to register as a new user.

Public \(\beta\) tools

Report a bug with a feature
Request a new functionality
404 page design
Send feedback

Attributions

(propose a free ad)

Site Statistics

205 submissions , 163 unreviewed
5,047 questions , 2,200 unanswered
5,345 answers , 22,709 comments
1,470 users with positive rep
816 active unimported users
More ...

  The contribution to the one loop beta function for the WZW model

+ 3 like - 0 dislike
563 views

When the Wess-Zumino-Witten model

$$S_{WZW}=\frac{k}{4\pi}\int d^2 z \, \, \mathrm{Tr}[\partial u \bar{\partial}u^{-1} ]+ \frac{k}{12\pi}\int d^3 \sigma \epsilon^{ijk}\, \mathrm{Tr}[(u^{-1}\partial_i u)(u^{-1}\partial_j u)(u^{-1}\partial_k u)]$$

is expanded around a solution of the equations of motions $u=u_0 e^{iT^a\pi^a}$ one gets

$$S_{WZW}=\frac{k}{4\pi}\int d^2 z \lbrace \, \mathrm{Tr}[\partial u_0 \bar{\partial}u_0^{-1} ]+ \frac{1}{2}\partial_\mu \pi^a\partial ^\mu \pi^a \\ +\frac{1}{2}(\eta^{\mu\nu}-\epsilon^{\mu\nu}) \, \mathrm{Tr}\lbrace (u_0^{-1}\partial_\mu u_0)[T^a\pi^a,T^b\partial_\nu \pi^b]\rbrace + \mathcal{O}(\pi^3)\rbrace$$

The one loop renormalization diagram is like

enter image description here

In "Non-Perturbative Field Theory" by Y.Frishman and J.Sonnenschein (chapter 4.2 page 65) I read that the non vanishing contributions come only from the diagrams with both vertices proportional to $\eta^{\mu\nu}$ or to $\epsilon^{\mu\nu}$. Could someone explain me how one comes to that conclusion?

This post imported from StackExchange Physics at 2014-12-09 15:13 (UTC), posted by SE-user Anne O'Nyme
asked Nov 28, 2014 in Theoretical Physics by Anne O'Nyme (175 points) [ no revision ]

1 Answer

+ 1 like - 0 dislike

One comes to this conclusion due to the fact that the contraction of a symmetric tensor with an antisymmetric one vanishes. Writing down the loop diagrams involves a contraction of both vertices. If you get expressions proportional to $\epsilon_{\mu\nu}\eta^{\mu\nu}$, this will be zero due to the fact that the metric is symmetric and the epsilon tensor is antisymmetric.

This post imported from StackExchange Physics at 2014-12-09 15:13 (UTC), posted by SE-user Frederic Brünner
answered Nov 28, 2014 by Frederic Brünner (1,130 points) [ no revision ]

Your answer

Please use answers only to (at least partly) answer questions. To comment, discuss, or ask for clarification, leave a comment instead.
To mask links under text, please type your text, highlight it, and click the "link" button. You can then enter your link URL.
Please consult the FAQ for as to how to format your post.
This is the answer box; if you want to write a comment instead, please use the 'add comment' button.
Live preview (may slow down editor)   Preview
Your name to display (optional):
Privacy: Your email address will only be used for sending these notifications.
Anti-spam verification:
If you are a human please identify the position of the character covered by the symbol $\varnothing$ in the following word:
p$\hbar$ysi$\varnothing$sOverflow
Then drag the red bullet below over the corresponding character of our banner. When you drop it there, the bullet changes to green (on slow internet connections after a few seconds).
Please complete the anti-spam verification




user contributions licensed under cc by-sa 3.0 with attribution required

Your rights
...